LSAT and Law School Admissions Forum

Get expert LSAT preparation and law school admissions advice from PowerScore Test Preparation.

 Administrator
PowerScore Staff
  • PowerScore Staff
  • Posts: 8917
  • Joined: Feb 02, 2011
|
#25171
Complete Question Explanation

Must Be True—PR. The correct answer choice is (D)

After hearing her favorite novelist speak out against the political candidate that she has supported for years, Sharon lowered her opinion of the novelist, but not of the political candidate. The question stem asks us to identify a principle to which this situation most closely conforms, and the correct answer choice will likely support Sharon’s behavior in broader terms: why did she lower her opinion of the novelist, but not of the politician?

Since a principle is by definition a broad rule (usually conditional in nature), the presence of the Principle indicator serves to broaden the scope of the question, which requires a more abstract understanding of the underlying relationships in the argument. In this problem, the principle must establish the conditions under which our positive opinion of someone might change for the worse. Sharon’s opinion did when she heard her favorite novelist speak out against a politician she had supported for years. Sharon probably shares the politician’s values and beliefs. So, if we hear someone speak out on political matters that contradict our long-standing beliefs, we will probably dismiss the speaker rather than change our beliefs. This prephrase should help you eliminate the four incorrect answer choices relatively quickly.

Answer choice (A): Sharon’s situation does not necessarily conform to this principle, because we do not know if she was among the novelist’s most dedicated fans. If she were, then this principle would actually justify the opposite outcome to the one described, because the novelist clearly did not influence Sharon’s beliefs.

Answer choice (B): This principle is entirely irrelevant to the situation described in the stimulus, because it is unclear if the novelist in question had established a reputation for being a “knowledgeable observer of politics.”

Answer choice (C): Having heard actors discuss issues they know nothing about, we might be tempted to agree with this principle. However, it has no bearing on the situation described in the stimulus. Sharon did not conclude that the novelist should “just shut up” about politics; she merely lowered her opinion of him while continuing to support the political candidate. This principle does not account for the outcome described, making answer choice (C) incorrect.

Answer choice (D): This is the correct answer choice. The novelist contradicted Sharon’s political beliefs, and as a result she lowered her opinion of him. So, generally speaking, if we hear a testimony that contradicts a long-standing belief (Sharon had supported the political candidate "for years"), we would question the source of the testimony (i.e. the novelist) rather than the correctness of the opinion (i.e. change our beliefs).

Answer choice (E): This answer choice may be attractive, because the principle correctly predicts that Sharon is unlikely to renounce her allegiance to the politician, whom she has supported for years. However, we cannot be sure if Sharon’s allegiance to the novelist is “new” to her: for all we know, this may have been her favorite novelist for decades. If so, then Sharon should not have renounced either allegiance – contrary to what actually happened. Since this principle does not conform to the situation as it is described in the stimulus, answer choice (E) is incorrect.
 glasann
  • Posts: 61
  • Joined: Jan 07, 2020
|
#91652
I correctly chose D on this and understand why it's right/others are wrong. However, I want to make sure I really understand these principle-conform question types.

Should I be treating it almost like a parallel question? Or, is it ok if the answer choice shows a scenario that is a little more BROAD than the situation in the stimulus, thereby ensuring that the situation in the stimulus is encompassed? Sometimes I'm wary of strong, overly broad/general answer choices (like about "all people", etc) unless it's a justify or strengthen question.

Thanks!
 Robert Carroll
PowerScore Staff
  • PowerScore Staff
  • Posts: 1787
  • Joined: Dec 06, 2013
|
#91664
glasann,

In this particular situation, the stimulus is a scenario and the answer choices are general principles that may (for the correct answer) or may not (wrong answers) cover the scenario in the stimulus. You want the stimulus to be a specific instance of the general principle in the answer choice. Almost by definition, the answer has to be broader than the stimulus - the stimulus is one case of the general rule, so the general rule is going to cover other things than just this situation. What you want to make sure is that the answer choice does apply to the stimulus situation - nothing is left "unknown". So, for instance, in answer choice (A), we don't know whether Sharon is one of the novelist's most dedicated fans, so the stimulus would be a poor instance of answer choice (A) - we have too much "unknown".

Robert Carroll

Get the most out of your LSAT Prep Plus subscription.

Analyze and track your performance with our Testing and Analytics Package.